Difference between revisions of "1987 AHSME Problems/Problem 30"
m (→Problem) |
(Added a solution with explanation) |
||
Line 25: | Line 25: | ||
\textbf{(D)}\ \frac{1}{\sqrt[3]{6}}\qquad | \textbf{(D)}\ \frac{1}{\sqrt[3]{6}}\qquad | ||
\textbf{(E)}\ \frac{1}{\sqrt[4]{12}} </math> | \textbf{(E)}\ \frac{1}{\sqrt[4]{12}} </math> | ||
+ | |||
+ | == Solution == | ||
+ | First we show that <math>E</math> is on <math>AC</math>, as in the given figure, by demonstrating that if <math>E = C</math>, then <math>\triangle ADE</math> has more than half the area, so <math>DE</math> is too far to the right. Specifically, assume <math>E = C</math> and drop an altitude from <math>C</math> to <math>AB</math> that meets <math>AB</math> at <math>F</math>. Without loss of generality, assume that <math>CF = 1</math>, so that <math>\frac{\text{Area} \ \triangle EAD}{\text{Area} \ \triangle EAB} = \frac{AD}{AB}</math> (as they have the same height, and area is <math>\frac{1}{2}</math> times the product of base and height), which is <math>\frac{1 + \frac{1}{\sqrt{3}}}{1 + \sqrt{3}} = \frac{1}{\sqrt{3}} > \frac{1}{2}</math>, as required. Thus we must move <math>DE</math> to the left, scaling <math>\triangle EAD</math> by a factor of <math>k</math> such that <math>\text{Area} \ \triangle EAD = \frac{1}{2} \ \text{Area} \ \triangle CAB \implies \frac{1}{2}k^{2}(1+\frac{1}{\sqrt{3}}) = \frac{1}{4}(1+\sqrt{3}) \implies k^{2} = \frac{\sqrt{3}}{2} \implies k = (\frac{3}{4})^{\frac{1}{4}}</math>. Thus <math>\frac{AD}{AB} = \frac{k(1+\frac{1}{\sqrt{3}})}{1+\sqrt{3}} = \frac{k}{\sqrt{3}} = (\frac{3}{4})^{\frac{1}{4}} \times (\frac{1}{9})^{\frac{1}{4}} = (\frac{1}{12})^{\frac{1}{4}} = \frac{1}{\sqrt[4]{12}}</math>, which is answer <math>\boxed{E}</math>. | ||
== See also == | == See also == |
Latest revision as of 12:17, 31 March 2018
Problem
In the figure, has and . A line , with on and , divides into two pieces of equal area. (Note: the figure may not be accurate; perhaps is on instead of The ratio is
Solution
First we show that is on , as in the given figure, by demonstrating that if , then has more than half the area, so is too far to the right. Specifically, assume and drop an altitude from to that meets at . Without loss of generality, assume that , so that (as they have the same height, and area is times the product of base and height), which is , as required. Thus we must move to the left, scaling by a factor of such that . Thus , which is answer .
See also
1987 AHSME (Problems • Answer Key • Resources) | ||
Preceded by Problem 29 |
Followed by Last Question | |
1 • 2 • 3 • 4 • 5 • 6 • 7 • 8 • 9 • 10 • 11 • 12 • 13 • 14 • 15 • 16 • 17 • 18 • 19 • 20 • 21 • 22 • 23 • 24 • 25 • 26 • 27 • 28 • 29 • 30 | ||
All AHSME Problems and Solutions |
The problems on this page are copyrighted by the Mathematical Association of America's American Mathematics Competitions.